Help I need an answer asap

Help I Need An Answer Asap

Answers

Answer 1

Answer:

6

Step-by-step explanation:

To find how many perennials there are, multiply the amount of plants by the percent of perennials. When you multiply percents, just put the percent behind the decimal

60x0.35=21

There are 21 perennials

Do the same for the biennials:

60x0.25=15

Therefore 15 biennials

Subtract the amount of perennials from the amount of biennials

21-15=6

There is 6 more perennials than biennials

Answer 2

Answer:

the answer is 6

21-15=6

Step-by-step explanation.

35% of 60 = 21 for perenalis

25% of 60 = 15 for biennials

40% of 60 = 24 for annuals


Related Questions

Could someone help me with 2, 9, 10, 11, 12 13, and 14?

Answers

Answer:

13.  (1) x + 8x = 32

14.  $337.5

Step-by-step explanation:

14. $270 x 0.25 = 67.5

270 + 67.5 = 337.5

I am sorry. I only got these-

yea 14 is 337.50 that’s all I got tho

b) Write 25 x 10^6 in standard form.​

Answers

Answer:

25000000

hope this helps

have a good day :)

Step-by-step explanation:

The standardised form is 25,000,000. I hope this helps!

What is the solution to the inequality below x2 > 64

Answers

answer:-8<x<8 step by step: (x^2) <64  => (x^2) -64 < 64-64 => (x^2) - 64 < 0 64= 8^2    so    (x^2) - (8^2) < 0 To solve the inequality we first find the roots (values of x that make (x^2) - (8^2) = 0 ) Note that if we can express (x^2) - (y^2) as (x-y)* (x+y)  You can work backwards and verify this is true. so let's set (x^2) - (8^2)  equal to zero to find the roots: (x^2) - (8^2) = 0   => (x-8)*(x+8) = 0       if x-8 = 0 => x=8      and if x+8 = 0 => x=-8 So x= +/-8 are the roots of x^2) - (8^2)Now you need to pick any x values less than -8 (the smaller root) , one x value between -8 and +8 (the two roots), and one x value greater than 8 (the greater root) and see if the sign is positive or negative. 1) Let's pick -10 (which is smaller than -8). If x=-10, then (x^2) - (8^2) = 100-64 = 36>0  so it is positive2) Let's pick 0 (which is greater than -8, larger than 8). If x=0, then (x^2) - (8^2) = 0-64 = -64 <0  so it is negative3) Let's pick +10 (which is greater than 10). If x=-10, then (x^2) - (8^2) = 100-64 = 36>0  so it is positive Since we are interested in (x^2) - 64 < 0, then x should be between -8 and positive 8. So  -8<x<8

Answer:

Step-by-step explanation:

x2

if logx √2 = 1/6
, then x =?

Answers

Step-by-step explanation:

logx√2=1/6

<=> logx(2^1/2)=1/6

<=>1/2.logx(2)=1/6

<=>logx(2)=1/3

<=>2=x^1/3

<=>x=

Sorry I forgot the formula in the last step

Answer:

v= 331m/s +(0.6 m/s /°C)×T

Given,

T=40°C

Substituting the value of T we get,

v=331m/s +(0.6 m/s /°C)×40°C

= 331m/s +(0.6 m/s)×40

= 331m/s+(24.0)

=355m/s

Application: Dana is planning to ride her bicycle around the circular race track and wants to know the distance. If the diameter of the track is 2 mile, how far is the distance around the race track?

Answers

Answer:

6.2831853071795864miles

~6.283

Step-by-step explanation:

Circumference of a circle=2πr

=2π×½×2mile

2×π×1mile

=6.2831853071795864miles

~6.283

Or, πd

=π×2miles

=6.2831853071795864miles

~6.283

The elimination method is ideal for solving this system of equations. By which number must you multiply the second equation to eliminate the y-variable, and what is the solution for this system?

Answers

The question is incomplete, the complete question is:

The elimination method is ideal for solving this system of equations. By which number must you multiply the second equation to eliminate the y-variable, and what is the solution for this system?

x + 3y = 42

2x − y = 14

A: Multiply the second equation by -3. The solution is x = 12, y = 9.

B: Multiply the second equation by -2. The solution is x = 12, y = 10.

C: Multiply the second equation by 2. The solution is x = 15, y = 9

D: Multiply the second equation by 3. The solution is x = 12, y = 10

Answer: The correct option is D.

Step-by-step explanation:

The elimination method is a technique wherein we eliminate the coefficient of any one variable.

The given equations are:

x + 3y = 42

2x − y = 14

We multiply the second equation by (3) and the equations formed are:

x + 3y = 42

6x − 3y = 42

The final equation after eliminating the y-term becomes:

7x = 84

x = 12

Putting value of 'x' in any of the original equation, we get:

⇒ 12 + 3y = 42

⇒ 3y = 30

⇒ y = 10

Hence, the correct option is D.

Solve for b. -11b+7 =40 what does b equal

Answers

Answer: b=-3

-11b+7=40
-7 -7
-11b=33
divide by -11 on both sides
b=-3

Explanation: In order to solve for x, you need to subtract 7 on both sides and then divide by -11 on both sides. You should get b=-3.

State the mathematical properties of all the steps in the equation below.

(2x + 3) − 7 = 8 + x Problem Statement.
(2x + 3) + (−7) = 8 + x _____________
((2x + 3) + (−7)) + 7 = (8 + x) + 7_____________
(2x + 3) + (−7 + 7) = (8 + x) +7 _____________
(2x + 3) + 0 = (8 + x) + 7 _____________
2x + 3 = (8 + x) + 7 _____________
2x + 3 = 7 + (8 + x) _____________
2x + 3 = (7 + 8) + x _____________
2x + 3 = 15 + x _____________
(2x + 3) + (−3) = (15 + x) + (−3) _____________
2x + (3 + (−3)) = (15 + x) + (−3) _____________
2x + 0 = (15 + x) + (−3) _____________
2x = (15 + x) + (−3) _____________
2x = (−3) + (15 + x) _____________
2x = (−3 + 15) + x _____________
2x = 12 + x _____________
2x + (−x) = 12 + x + (−x) _____________
2x + (−x) = 12 + 1 · x + (−x) _____________
2x + (−x) = 12 + 1 · x + (−1) · x _____________
2x + (−x) = 12 + (1 + (−1))x _____________
2x + (−x) = 12 + 0 · x _____________
2x + (−x) = 12 + 0 _____________
2x + (−x) = 12 _____________
2x + (−1) · x = 12 _____________
(2 + (−1)) · x = 12 _____________
1 · x = 12 _____________
x = 12 Solution

Answers

Answer:

173*3*j3

Step-by-step explanation:

naksbvajahagcgaja

Answer:

HIHHIHIHIHIHIHIHIH

Step-by-step explanation:

IHIHIHIHIHIHIHIHIH

what is the value of x?

Answers

Answer:

6 sqrt(3) = x

Step-by-step explanation:

Since this is a right triangle, we can use trig functions

sin theta = opp/ hyp

sin 60 = x/12

12 sin 60 =x

12 ( sqrt(3)/2) =x

6 sqrt(3) = x

Answer:

x=6[tex]\sqrt{3}[/tex]

Step-by-step explanation:

take 60 degree as reference angle

using sin rule

sin 60=opposite/hypotenuse

[tex]\sqrt{3[/tex]/2=x/12

do cross multiplication

2*x=12*[tex]\sqrt{3}[/tex]

x=12[tex]\sqrt{3}[/tex] /2

x=6[tex]\sqrt{3}[/tex]

Find the value of x.​

Answers

Answer:

[tex]x=4[/tex]

Step-by-step explanation:

The two given triangles, triangle (BDE) and triangle (BAC) share the angle (<B) in common. Moreover, their bases, line (DE), and line (AC) are parallel, this means that through the corresponding angles postulate, angles (<BDE) and (<BAC) are congrunet. Moreover, angles (<BED) and (<BCA) are also congruent. Thus, triangles (BDE) and (BAC) are similar through angle-angle similarity.

The ratio between the corresponding sides of similar triangles is the same. Therefore, one can form the following equation,

[tex]\frac{BD}{BA}=\frac{BE}{BC}[/tex].

Substitute,

[tex]\frac{BD}{BD+DA}=\frac{BE}{BE+EC}\\\\\frac{x+2}{x+x+2}=\frac{3}{3+2}[/tex]

Simplify,

[tex]\frac{x+2}{2x+2}=\frac{3}{5}[/tex]

Cross products,

[tex]5(x+2)=3(2x+2)\\[/tex]

Simplify, distribute, multiply every term inside the parenthesis by the term outside of it,

[tex]5x+10=6x+6[/tex]

Inverse operations,

[tex]5x+10=6x+6\\\\10=x+6\\\\4=x[/tex]

8.
Which quadratic function is shown in the graph below?

Answers

The answer is (C) y = 1/2x^2

Math help please thanks

Answers

Answer:

x = 10.5

Step-by-step explanation:

The first thing you have to do is to find out what you multiplied (say) 2 by to get 3.5 and then try it again with another number.

2k = 3.5         Divide by 2

k = 3.5 / 2

k = 1.75

Now try it on another number. What do you multiply 4 by to get 7

4k = 7            Divide by 4

k = 7/4

k = 1.75

So here's what's going on. If you multiply every dimension you are given on the left pentagon (it's not regular) by 1.75, you should get the corresponding number on the right.

To get x you need to multiply 6 by 1.75

x = 6*1.75

x = 10.5

A pencil company packs pencils in boxes of 8. How many boxes can they pack with 32,000 pencils? *


A 40,000
B 4,000
C 400
D 40

Answers

Answer:

B: 4,000

Step-by-step explanation:

32000/8=4000

Answer:

4000

Step-by-step explanation:

32000/8............

pa help po............. ............​

Answers

Answer:

ΔABC

∠9 = ∠19 + ∠21

∠7 = ∠20 + ∠21

∠8 = ∠19 + ∠20

ΔXYZ

∠3 = ∠11 + ∠12

∠5 = ∠10 + ∠12

∠4 = ∠10 + ∠11

ΔNOR

∠1 = ∠5 + ∠6

∠2 = ∠ 4 + ∠5

∠3 = ∠4 + ∠6

Step-by-step explanation:

According to the exterior angle theorem, the exterior angle of a triangle is equal in measurement to the sum of the two interior angles of the triangle;

In triangle ΔABC, we have;

∠9 = ∠19 + ∠21

∠7 = ∠20 + ∠21

∠8 = ∠19 + ∠20

In triangle ΔXYZ, we have;

∠3 = ∠11 + ∠12

∠5 = ∠10 + ∠12

∠4 = ∠10 + ∠11

In triangle ΔNOR, we have;

∠1 = ∠5 + ∠6

∠2 = ∠ 4 + ∠5

∠3 = ∠4 + ∠6

Martha has two pet poodles. Fifi, her standard poodle, weighs 7 times as much as
Buttercup, her toy poodle.
Pick the diagram that models the ratio in the story.
Fifi
Buttercup
Fifi
Buttercup
If Fifi weighs 56 pounds, how much does Buttercup weigh?
pounds

Answers

Answer:

Buttercup weighs 8 pounds

Step-by-step explanation:

Sorry I can't pick a diagram, since we can't see a photo.

if b is Buttercup and f is Fifi, substitute in Fifi's weight, and solve for b

b x 7 = f

7b = 56

divide both sides by 7

b = 8

have no idea how to do this

Answers

Answer:

ans: option C is correct answer

Step-by-step explanation:

follow steps as shown in picture above,

Factorise completely the expression below: 6ax+12by-9ay-8bx​

Answers

Answer:

Step-by-step explanation:

6ax +12by -9ay -8bx

=6ax -8bx -9ay +12by

=2x(3a -4b)-3y(3a -4b)

=(2x-3y)(3a-4b)

At 6 a.m., the temperature was –8°C. By noon, the temperature had risen 6°C. The temperature at 6 p.m. was –10°C. Which integer represents the temperature change between noon and 6 p.m.?
–24
–12
–8
–4

Answers

Answer:

-8

Step-by-step explanation:

so -8 + 6 = -2. and then for that to drop to -10, it would have to change -8 degrees

Answer:

Step-by-step explanation:

-8 is the answer

11. It refers to the information that supports the claim.
A. reference
B. evidence
C. cite
D. issue​

Answers

Answer:

B. Evidence

Step-by-step explanation:

Reference: like when your drawing something and you want to make sure that you draw the hair the same way some other person did.

Cite: That’s when you give the credits to someone or something 90% sure

Hope this helps :)

Area of the grass area only:

Area of the track only:

Answers

Answer:

Step-by-step explanation:

area including track=area of 2 semi circles+area of rectangle

=2×1/2×π×46²+86×65

=2116 π+5590

area of grass=2×1/2×π×40²+80×65=1600π+5200

≈10226.55 m²

area of track=2116π+5590-1600π-5200

=516π+390

≈2011.06 m²

What is the slope of the line shown?

Answers

Answer:

1/2

Step-by-step explanation:

Write the equation of a line perpendicular to y = -5x + 1 that goes through (10. 4).
Show all work.

Answers

Answer:

y = 1/5x + 2

Step-by-step explanation:

y = 1/5x + b

4 = 1/5(10) + b

4 = 2 + b

2 = b

A salesman is paid $450 a week, plus a commission of 35 cents for every item that he sells. Write a linear function where the paycheck, P, of the salesman is a function of the number, N, of items he sells. If the salesman only has 1400 items available to sell each week, find the domain and range of the function.

Answers

Answer:

Step-by-step explanation:

1400×0.35

490+450

940

Answer:

The domain is thus [0, 1400].

The range is thus [$450, $940]

Step-by-step explanation:

A suitable function is P(N) = ($450) + ($0.35 / item)*N, where $450 is the base pay per week and N is the number of items sold.

In this case the salesman has only 1400 items available to sell each week.  Therefore N begins at 0 (no items to sell) and ends at 1400 (the max number of items available to sell).  The domain is thus [0, 1400].

The smallest amount the salesman could receive would be when he has not sold any items:  $450.  

The max amount is then $450 + ($0.35/item)(1400 items), or

                                         $450 + $490, or

                                          $940

The range is thus [$450, $940]

A student recorded the following distance measurements 4.3 miles ,1.7 miles and 5.6 miles what are the total measurements recorded

Answers

Answer:

11.6 miles

Step-by-step explanation:

4.3 miles + 1.7 miles + 5.6 miles = (4.3+1.7+5.6)miles

= 11.6 miles

Given the graph of the line, choose two points and find the slope. Construct the equations for each of the
points you chose in point slope form. Show your work and explain each step.
*The graph is in the picture *
PLEASE HELP I WILL GIVE BRAINLIST

Answers

Step-by-step explanation:

First, given that we must use point slope form, we can define that as

y - y₁ = m (x-x₁), with m being the slope .

To find the slope, we can use the equation

[tex]\frac{y_2 - y_1}{x_2 - x_1}[/tex]. Two points on the graph are (0, 1) and (1, 3). For these points, when calculating the slope, 0 and 1 represent x₁ and y₁ respectively, while 1 and 3 represent x₂ and y₂ respectively Using this formula, we can plug our points in to get

[tex]\frac{3-1}{1-0} = 2/1 = 2[/tex]

as our slope. Therefore, our equation is

y - y₁ = 2 (x-x₁).

For our first point, (0,1), we can simply plug 0 in for x₁ and 1 in for y₁ to get

y - 1 = 2(x-0) as one equation

Next, for (1,3) we can plug 1 for x₁ and 3 for y₁ to get

y - 3 = 2 (x-1) as our other equation

A 12-foot ladder is leaning against a wall. The distance from the base of the wall to the base of the ladder is 6 StartRoot 2 EndRoot feet. Given this information, what can be determined about the triangle formed by the ground, wall, and ladder? Check all that apply.

Answers

Answer:

a

b

d

e

Step-by-step explanation:

he triangle is isosceles.

The leg-to-hypotenuse ratio is 1:StartRoot 2 EndRoot.

The leg-to-hypotenuse ratio is 1:StartFraction StartRoot 2 EndRoot Over 2 EndRoot. The nonright angles are congruent. The ladder represents the longest length in the triangle.

Answer:

a,b,d,e

Step-by-step explanation:

answer

Please help…………………………….

Answers

if i'm correct its 7x

Answer:

Domen is [ -2 , infinitely) or can be written as

-2 < x < infinitely in inequality form.

Step-by-step explanation:

if (x+2) is negative than function will be imaginary so to be real function (x+2) should be positive

hence answer is : Domen = -2 < x < infinitely or can be written as [ -2 , infinitely )

If K(4,-5) is rotated counter clockwise 270°, what will be the coordinates of its image?
K'(5,-4)
K'(5, 4)
K'(-4,5)
K'(-5,-4)

Answers

Answer:

(-5, -4)

Step-by-step explanation:

The formula for a point (x, y) rotating 270 degrees counterclockwise is that it turns into (y, -x). Thus, for (4, -5), its image would be (-5, -4)


Use SOH CAH TOA to find the missing side.

Answers

sin(43) × hypotenuse of 19 gives you oppo

= 13

help with the question below please!!

Answers

Answer:

a.= 24 b.=-21 c. is x=-7 or 3

Step-by-step explanation:

Step-by-step explanation:

a) f(5)= 5² + 4 x 5 -21

=24

b) f(0)= 0² + 4 x 0 -21

= -21

c) f(x)= 0

=> x² + 4x -21 =0

=> x = 3, x= -7

Other Questions
1.Round 3 601 111 to the nearest million2. Round 17.81 to the nearest whole number3. Round 59.52 to the nearest 1 d.p.4. Round 7.176 to the nearest 2d.p.(Pls help me) The radius of a circle is 5 feet. Help with math 10 points Science is all around you. Sort the ways you are affected by science. Move the tiles to show whether each idea isrelated to the weather or to technology.turning on the lightsRelated to the WeatherRelated to Technologydressing for the forecastseeing a flag blowriding in a car or busdownloading a songpreparing for a hurricane What is the total SURFACE AREA of a cylinder with a radius of 5 cm and a height of 7 cm? Pls Help I'm really really stumped. WILL MARK YOU IF YOU CAN HELP plz help me understand this to or add one of my socials to help me with geometry:(Insta:aesthetictoxicbabe Snap:Toxicangelbby The fate of most of the Okies and other Dust Bowl migrants who headed west to California was that they a found steady work in the canning industry. b acquired farms in the San Joaquin Valley. c formed mutually supportive evangelical religious communes. d joined reactionary, quasi-fascist organizations in protest of Roosevelt's economic policies. e found themselves mired in poverty, squalor, and lack of economic opportunity in the San Joaquin Valley The price of a mobile phone was 980.In a sale the price is reduced by 25%.On the final day the sale price is reduced by a further 8%.How much is saved in total by buying the mobile phone on the final dayof the sale? How many moles are in 243 grams of Al(OH)3? Help me please anyone :' )What do you believe to be the Historical Significance of the Persian Wars? Battle of Marathon, Thermopylae, and Plataea. I have to write a paragraph (6 ish sentences), so if you can do that that would be great. If you can't, please give me some good points that I can add! I will give brainliest and NO LINKS! Which population of organisms would be in greatest danger of becoming extinct? *A. A population of organisms having few variations living in a stable environment.B. A population of organisms having many variations living in a stable environment.C. A population of organisms having few variations living in an unstable environment.D. A population of organisms having many variations living in an unstable environment. CAN SOMEONE HELP ME I HAVE LIKE LESS THAN 15 MINUTES TO FINISH Mannisto Inc. uses the FIFO inventory cost flow assumption. In a year of rising costs and prices, the firm reported net income of $219,017 and average assets of $1,413,720. If Mannisto had used the LIFO cost flow assumption in the same year, its cost of goods sold would have been $36,220 more than under FIFO, and its average assets would have been $31,640 less than under FIFO.Required:Calculate the firm's ROI under each cost flow assumption (FIFO and LIFO). Finding the volume of the cone Who at the top of the food web 20 divided by 34 HEEEEELPPPPPPPPPPP ubuhle nobubi bokuvakasha ngamaholidi essay I will give anyone who can give me the right answer to the Brainist! I really need an answer! Find the missing dimension of the prism. Round your answer to the nearest hundredth.Volume = 72.66 in.?2 in.